UPSC Civil Services Main 1990 - Mathematics Calculus: Sunder Lal
UPSC Civil Services Main 1990 - Mathematics Calculus: Sunder Lal
Calculus
Sunder Lal
Retired Professor of Mathematics
Panjab University
Chandigarh
Question 1(a) If a function f (x) of the real variable x has the first five derivatives 0 at a
given value x = a, show that it has a maximum or a minimum at x = a according as the
sixth derivative is negative or positive. What happens if only the first four derivatives are 0,
but not the fifth?
Solution. We shall take up the general case, which would cover both the cases
Let f (n) (a) exist and f (n) (a) 6= 0. Let f (r) (a) = 0, 1 r n 1. Then f (a) is not an
extreme value when n is odd, and if n is even, then f has a maximum at x = a if f (n) (a) < 0
and a minimum if f (n) (a) > 0.
Thus the above result is proved when n = 6. When n = 5, f (x) has neither maximum
nor minimum at x = a.
Proof: The existence of f (n) (a) imlies that f 0 (x), f 00 (x), . . . , f (n1) (x) all exist in a certain
neighborhood of a, say (a 1 , a + 1 ).
Case (1): If f (n) (a) > 0, then f (n1) (x) is increasing at a i.e. there exists 0 < < 1
such that f (n1) (x) < f (n1) (a) = 0 for a < x < a, and f (n1) (x) > f (n1) (a) = 0 for
a < x < a + .
Case (2): If f (n) (a) < 0, then f (n1) (x) is decreasing at a i.e. there exists 0 < < 1
such that f (n1) (x) > f (n1) (a) = 0 for a < x < a, and f (n1) (x) < f (n1) (a) = 0 for
a < x < a + .
Let h be such that |h| < , then by Taylors theorem
hn1 (n1)
f (a + h) = f (a) + hf 0 (a) + . . . + f (a + h), 0 < < 1
(n 1)!
hn1 (n1)
Thus f (a + h) f (a) = f (a + h) as f (r) (a) = 0, 1 r n 2. Since
(n 1)!
a + h (a , a + ), we have the following conclusions:
1
1. n even and f (n) (a) > 0: If h < 0, then hn1 < 0 and f (n1) (a + h) < 0 (by Case
(1) above, as a < a + h < a. If h > 0, then hn1 > 0 and f (n1) (a + h) > 0 (as
a < a + h < a + .
hn1 (n1)
In either case f (a + h) > 0 for all h with |h| < , i.e. f (a + h) > f (a)
(n 1)!
for |h| < , thus f (x) has a minimum at x = a.
hn1 (n1)
2. n even and f (n) (a) < 0: Using Case (2), we get f (a + h) < 0 for all h
(n 1)!
with |h| < , i.e. f (a + h) < f (a) for |h| < , thus f (x) has a maximum at x = a.
3. n odd and f (n) (a) > 0: If h < 0, then hn1 > 0 and f (n1) (a + h) < 0, so f (a +
h) < f (a) for h < 0, |h| < . If h > 0, then hn1 > 0 and f (n1) (a + h) > 0, so
f (a + h) > f (a) for 0 < h < . Thus f has neither minimum nor maximum at x = a.
The case for n odd and f (n) (a) < 0 is similar h < 0 f (a + h) > f (a), h > 0
f (a + h) < f (a), so there is no extreme value at x = a.
Solution.
f 0 (x) = 2(x 2)(x2 + 2bx + c)(x + 3)3 + (x 2)2 (2x + 2b)(x + 3)3
+ 3(x 2)2 (x2 + 2bx + c)(x + 3)2
f 0 (1) = 2(3)(1 2b + c)8 + 72(2 + 2b) + 108(1 2b + c)
= 48 + 96b 48c 144 + 144b + 108 216b + 108c = 84 + 24b + 60c
Question 2(b) Assuming that the above condition is satisfied, examine the nature of f (x)
at its three critical points.
Solution.
2
1. x = 2.
(x 2)2 (x2 + 2bx + c)(x + 3)3 = (x 2)2 g(x)
f (x) =
f 0 (x) = 2(x 2)g(x) + (x 2)2 g 0 (x)
f 00 (x) = 2g(x) + 4(x 2)g 0 (x) + (x 2)2 g 00 (x)
f 00 (2) = 2g(2) = 250(4 + 4b + c)
= 250(4 + 14 10c + c) = 250 9(2 c)
There is a maximum at x = 2 if 2 < c, and a minimum at x = 2 if 2 > c. The test
fails when c = 2. Considering f 000 (x) when x = 2, c = 2, 2b = 3,
f 000 (x) = 2g 0 (x) + 4g 0 (x) + 4(x 2)g 00 (x) + (x 2)2 g 000 (x) + 2(x 2)g 00 (x)
f 000 (2) = 6 (4 + 2b)(2 + 3)3 + 3(4 + 4b + c)(2 + 3)2
= 6 125 + 75(4 6 + 2) 6= 0
so there is no maximum or minimum at x = 2 when c = 2.
2. x = 3
(x + 3)3 (x 2)2 (x2 + 2bx + c) = (x + 3)3 h(x)
f (x) =
f 0 (x) = 3(x + 3)2 h(x) + (x + 3)3 h0 (x)
f 00 (x) = 6(x + 3)h(x) + 3(x + 3)2 h0 (x) + 3(x + 3)2 h0 (x) + (x + 3)3 h00 (x)
f 00 (3) = 0
f 000 (x) = 6h(x) + 12(x + 3)h0 (x) + (x + 3)2 [some polynomial in x]
f 000 (3) = 6h(3) = 150(9 6b + c) = 150(9 21 + 15c + c) = 600(4c 3)
So if c 6= 34 , f (x) does not have a maximum or minimum at x = 3. If c = 34 , then
3
So if c > 65 , then f 00 (1) < 0 and f (x) has a maximum at x = 1. If c < 65 , f (x) has
a minimum at x = 1. If c = 56 , then this test fails, we consider higher derivatives at
y = 2 with c = 65 :
Question 2(c) Show that f (x) has at least three points of inflection irrespective of the
condition 2b + 5c = 7.
n
where f (m) denotes the m-th derivative of f (x) and i
are the binomial coefficients.
4
Differentiating both sides,
m
(m+1)
X m
(f g) (x) = f (mi+1) (x)g (i) (x) + f (mi) (x)g (i+1) (x)
i=0
i
m h
m (m+1) (0)
X m m i (mi+1)
= f (x)g (x) + + f (x)g (i) (x)
0 i=1
i i 1
m (0)
+ f (x)g (m+1) (x)
m
m
m + 1 (m+1) (0)
X m + 1 (mi+1)
= f (x)g (x) + f (x)g (i) (x)
0 i=1
i
m + 1 (0)
+ f (x)g (m+1) (x)
m+1
m+1
X m + 1
= f (m+1i) (x)g (i) (x)
i=0
i
We used m0 = 1 = m+1
m m+1
m m
m+1
0
, m
= 1 = m+1
, i
+ i1
= i
. The result is now
established for n = m + 1, and hence by induction for all n.
Question 3(b) Show that the m-th derivative gm (x) of g(x) = tan1 x satisfies
1
Solution. g1 (x) = or (1 + x2 )g1 (x) = 1.
1 + x2
Using Leibnitzs theorem proved above and differentiating the above equation m times,
we get
m 2 m m
(1 + x )gm+1 (x) + 2xgm (x) + 2gm1 (x) = 0
0 1 2
or
(1 + x2 )gm+1 (x) + 2mxgm (x) + m(m 1)gm1 (x) = 0
which is the required relation.
1 (1)11 (1 1)!1 (x)
Now g1 (x) = = as 1 (x) = 1.
1 + x2 (1 + x2 )1
5
2x (1)21 (2 1)!2 (x)
g2 (x) = = as 2 (x) = 2x.
(1 + x2 )2 (1 + x2 )2
Assume that the result is true for all n m. From the relation proved above:
as m+1 (x) = 2xm (x) (1 + x2 )m1 (x). This completes the proof.
Question 3(c) Find m (x) for x 6. Can you find a general formula?
Solution.
1 (x) = 1
2 (x) = 2x
3 (x) = 2x 2x (1 + x2 ) = 3x2 1
4 (x) = 2x(3x2 1) (1 + x2 )(2x) = 4x3 4x
5 (x) = 2x(4x3 4x) (1 + x2 )(3x2 1) = 5x4 10x2 + 1
6 (x) = 2x(5x4 10x2 + 1) (1 + x2 )(4x3 4x)
= 6x5 20x3 + 6x
6
5 4 5 2 5 0
5 (x) = x x + x
4 2 0
6 5 6 3 6 1
6 (x) = x x + x
5 3 1
n n1 n n3 n
n (x) = x x + xn5 . . .
n1 n3 n5
2r+1n 2r+1n
X
r n n2r1
X
r n
= (1) x = (1) xn2r1
r=0
n 2r 1 r=0
2r + 1
Thus n n
dn x
1
X n X n
(e tan x) = g r (0) = (1)r1 (r 1)!r (0)
dxn x=0 r=0
r r=1
r
as g(0) = 0 and gr (x) was computed above in terms of r (x). Now 2r (0) = 0, and 2r+1 =
(1)r . Thus
2r+1n
dn x
X n
1
(e tan x) = (2r)!(1)r
dxn x=0 r=0
2r + 1
or
dn x
1 n n n n
n
(e tan x) = 0! 2! + 4! 6! + ...
dx x=0 1 3 5 7
as required.
Paper II
Z
2
Question 4(a) Discuss the convergence of the integral log(sin x) dx and if convergent,
0
evaluate it.
7
Solution. The integrand log sin x has a discontinuity only at x = 0. We consider the
Z
2
integral log(sin x) dx so that the integrand is positive.
0
Let g(x) = xm , 0 < m < 1, then
By putting 2x = t.
Z Z
2 1
log(sin 2x) dx = log(sin t) dt
0 2 0
Now substituting t = y, we have
Z Z 0 Z
2
log(sin t) dt = log(sin y) (dy) = log(sin y) dy
2 2
0
Z Z
2 2
log(sin 2x) dx = log(sin y) dy
0 0
Z
2
log(sin x) dx = log 2
0 2
2
Question 4(b) Find the point on the parabola
y = 2x, z = 0 which is nearest to the plane
z = x + 2y + 8. Show that this distance is 6.
Solution. Let the point (x, y, 0) by on the parabola. Then the distance d from the plane
z = x + 2y + 8 is given by
|x + 2y 0 + 8| |x + 2y + 8|
d= =
1+4+1 6
8
6d2 = x2 + 4xy + 4y 2 + 16x + 32y + 64.
Put 2x = y 2 , and let F = 6d2 to get
y4 2
F = 6d = + 2y 3 + 4y 2 + 8y 2 + 32y + 64
4
We need to minimize F . The critical points are given by
dF
= y 3 + 6y 2 + 24y + 32 = 0
dy
(y + 2)(y 2 + 4y + 16) = 0
d2 F
= 3y 2 + 12y + 24 = 12 24 + 24 > 0
dy 2
at y = 2. Hence y = 2 is a minimum.
When y = 2, x = 2, and d = |24+8|
6
= 6 as required.
x2 y 2 z 2 2 a2 b 2 c 2
over the volume bounded by 2 + 2 + 2 = 1 is equal to .
a b c 4
Solution. Let
ZZZ
1
I = 2
(a2 b2 c2 b2 c2 x2 c2 a2 y 2 a2 b2 z 2 ) 2 dx dy dz
x2 2
{(x,y,z) | + y2 + z2 1}
a2 b c
x2 y 2 z 2 21
ZZZ
= 8abc 1 2 2 2 dx dy dz
{(x,y,z) | x2 2 2
+ y2 + z2 1,x0,y0,z0} a b c
a2 b c
du dv dw
Let X 2 = u, Y 2 = v, Z 2 = w, so that dX = , dY
2 u
= , dZ
2 v
=
2 w
and
ZZZ
1 1 1 1
2 2 2
I=a b c (1 u v w) 2 u 2 v 2 w 2 du dv dw
u+v+w1
u0,v0,w0
9
We now convert this to a Dirichlet integral let u + v + w = , v + w = , w = ,
(u, v, w)
so that = 2 and
(, , )
Z 1Z 1Z 1
1 1 1 1
I = abc 2 2 2
(1 ) 2 ( ) 2 ( ) 2 () 2 2 d d d
Z0 1 Z0 1 Z0 1
1 1 1 1 1
= a2 b 2 c 2 (1 ) 2 2 (1 ) 2 (1 ) 2 2 d d d
0 0 0
3 3
2 2 2 ( 2 )( 2 ) ( 21 )(1) ( 21 )( 21 )
= abc
(3) ( 32 ) (1)
1 1 4
2 2 2 2 ( 2 ) 2 a2 b 2 c 2
= abc =
2 4
as required, as ( 12 ) = .
10